Show eigenvalues of hermitian operator are real

In summary: Yeah, I think I might have done an illegal assumption or operation. I wasn't 100% satisfied with what I had earlier. I don't remember what I put, but it should have been something like this\langle \phi | (AB)^\dagger | \psi \rangle = \langle (AB) \phi | \psi \rangle = \langle B \phi | A^\dagger \psi \rangle = \langle \phi | B^\dagger A^\dagger \psi \rangle
  • #1
Shackleford
1,656
2
http://i111.photobucket.com/albums/n149/camarolt4z28/2010-10-20165642.jpg?t=1287612122

http://i111.photobucket.com/albums/n149/camarolt4z28/2010-10-20165727.jpg?t=1287612136

Thanks.
 
Last edited by a moderator:
Physics news on Phys.org
  • #2
Those look fine. In the first one you've assumed orthonormality. You could have also noted that
[tex]\langle \psi_m | A | \psi _n \rangle = a_n \langle \psi_m | \psi_n \rangle[/tex]
[tex]\langle \psi_m | A^\dagger | \psi _n \rangle = a^*_m \langle \psi_m | \psi_n \rangle[/tex]
which when subtracted leads to
[tex](a_n-a_m^*)\langle \psi_m | \psi_n \rangle =0 [/tex]
This shows that the conjugate eigenvalue must actually be real, and that the states must be orthonormal.

For the second one I don't know what equations the problem tells you to use, so I'm not sure if it's okay or not.
 
  • #3
Mindscrape said:
Those look fine. In the first one you've assumed orthonormality. You could have also noted that
[tex]\langle \psi_m | A | \psi _n \rangle = a_n \langle \psi_m | \psi_n \rangle[/tex]
[tex]\langle \psi_m | A^\dagger | \psi _n \rangle = a^*_m \langle \psi_m | \psi_n \rangle[/tex]
which when subtracted leads to
[tex](a_n-a_m^*)\langle \psi_m | \psi_n \rangle =0 [/tex]
This shows that the conjugate eigenvalue must actually be real, and that the states must be orthonormal.

For the second one I don't know what equations the problem tells you to use, so I'm not sure if it's okay or not.

I fumbled through the first part. Your method is more concise, though. However, I'm not sure why am is conjugated. From the definition, when you operate the hermitian operator [tex]A^\dagger [/tex] on the left, it simply goes back to A in the Dirac notation. But in the integral form, I can see why the conjugation is there. In Dirac notation, is it there because the bra is the conjugate of the ket and so the conjugation of the operator is implied?

Here are the equations. I didn't really use them all that much. Should I try to do so?

http://i111.photobucket.com/albums/n149/camarolt4z28/2010-10-21082445.jpg?t=1287667815
 
Last edited by a moderator:
  • #4
Right, the ket conjugates the eigenvalue by definition of the notation.

For the second question you assumed that A and B commute, which is not true in general. Try a different approach, try just using the dirac notation to get where you want to go. Start with

[tex]\langle \phi | (AB)^\dagger | \psi \rangle = \langle \psi | A B | \phi \rangle^*[/tex]

and see where it takes you. You could even use integral form if you really wanted, but I, personally, like the dirac notation because it's concise.
 
  • #5
Mindscrape said:
Right, the ket conjugates the eigenvalue by definition of the notation.

For the second question you assumed that A and B commute, which is not true in general. Try a different approach, try just using the dirac notation to get where you want to go. Start with

[tex]\langle \phi | (AB)^\dagger | \psi \rangle = \langle \psi | A B | \phi \rangle^*[/tex]

and see where it takes you. You could even use integral form if you really wanted, but I, personally, like the dirac notation because it's concise.

Okay. Let me try that. If you want, check my other thread, too. That's the next problem I'm going to work on.

https://www.physicsforums.com/showthread.php?t=440132
 
  • #6
Mindscrape said:
Right, the ket conjugates the eigenvalue by definition of the notation.

For the second question you assumed that A and B commute, which is not true in general. Try a different approach, try just using the dirac notation to get where you want to go. Start with

[tex]\langle \phi | (AB)^\dagger | \psi \rangle = \langle \psi | A B | \phi \rangle^*[/tex]

and see where it takes you. You could even use integral form if you really wanted, but I, personally, like the dirac notation because it's concise.

http://i111.photobucket.com/albums/n149/camarolt4z28/2010-10-21221122.jpg?t=1287717351
 
Last edited by a moderator:
  • #7
Um, yeah, it looks like you're there.
 
Last edited:
  • #8
Mindscrape said:
Um, yeah, it looks like you're there.

I noticed you took out your work. Any reason for it?
 
  • #9
Yeah, I think I might have done an illegal assumption or operation. I wasn't 100% satisfied with what I had earlier. I don't remember what I put, but it should have been something simple like this

[tex]
\langle \phi | (AB)^\dagger | \psi \rangle = \langle (AB) \phi | \psi \rangle = \langle B \phi | A^\dagger \psi \rangle = \langle \phi | B^\dagger A^\dagger \psi \rangle
[/tex]
Step 1 uses changing between bras and kets, step 2 uses the adjoint to move B phi, and step 3 uses bra ket equivalence again.
 
  • #10
Mindscrape said:
When I looked back at your solution it wasn't a whole lot different from mine, so I didn't think I needed to reiterate. I don't remember what I put, but it should have been something like this

[tex]
\langle \phi | (AB)^\dagger | \psi \rangle = \langle (AB) \phi | \psi \rangle = \langle B \phi | A^\dagger \psi \rangle = \langle \phi | B^\dagger A^\dagger \psi \rangle
[/tex]

Oh, okay. Thanks for the help so far. I'm trying to do my best to do the homework problems and understand everything well enough so I can do better on the next exam.

Could you help me on the next problem or two? I don't know why I'm having trouble. It looks like a fairly straightforward problem. :frown:

https://www.physicsforums.com/showthread.php?t=440132
 
  • #11
Well, I'm reviewing my notes for the exam on Wednesday. I got full-credit for this problem. However, I don't understand how the last line is valid. The psi and phi should be reversed since I'm taking the hermitian conjugate of a conjugate.

http://i111.photobucket.com/albums/n149/camarolt4z28/2010-10-21221122.jpg?t=1287717351
 
Last edited by a moderator:
  • #12
Actually, now that I look back on it, yeah, all your work would have done would have been to take you in a circle back to [itex](AB)^\dagger[/itex]. If you understand the solution I put up, go with this solution since it's pretty concise and really just uses bra-ket formalism. Maybe both the grader and I just got lazy before and didn't fully check your work.
 
  • #13
Mindscrape said:
Actually, now that I look back on it, yeah, all your work would have done would have been to take you in a circle back to [itex](AB)^\dagger[/itex]. If you understand the solution I put up, go with this solution since it's pretty concise and really just uses bra-ket formalism. Maybe both the grader and I just got lazy before and didn't fully check your work.

Yeah, I understand what you did. The book actually proves it like that, too. I overlooked that part when doing the homework.

I'm not actually sure how to interpret what I wrote since there are a few different ways to write out this stuff.

Would it be <phi /(AB)^+/psi>^+ = <psi /(AB)/phi> ?
 
  • #14
Yeah, that's what you would get.
 

1. What is a Hermitian operator?

A Hermitian operator is a linear operator in a complex vector space that satisfies the condition of being equal to its own adjoint. In other words, the operator and its adjoint have the same matrix representation.

2. Why are the eigenvalues of a Hermitian operator real?

The eigenvalues of a Hermitian operator are real because of the Hermitian property, which states that the operator is equal to its own adjoint. This means that the operator and its adjoint have the same eigenvalues, and since the adjoint of a Hermitian operator is the complex conjugate of the operator, the eigenvalues must also be complex conjugates. Therefore, the only way for the eigenvalues to be equal to their complex conjugates is if they are real numbers.

3. How is the Hermitian property related to real eigenvalues?

The Hermitian property is the key reason why the eigenvalues of a Hermitian operator are real. This property ensures that the operator and its adjoint have the same eigenvalues, and the adjoint of a Hermitian operator is the complex conjugate of the operator. This means that the eigenvalues must also be complex conjugates, which can only be true if they are real numbers.

4. Can a non-Hermitian operator have real eigenvalues?

No, a non-Hermitian operator cannot have real eigenvalues. The Hermitian property is a necessary condition for the eigenvalues of an operator to be real. Without this property, the operator and its adjoint will not have the same eigenvalues, and therefore, the eigenvalues will not be real.

5. How are real eigenvalues of a Hermitian operator different from complex eigenvalues?

The real eigenvalues of a Hermitian operator are different from complex eigenvalues in that they do not have a complex component. This means that they lie on the real number line. On the other hand, complex eigenvalues have both a real and imaginary component and lie in the complex plane. The Hermitian property ensures that the eigenvalues of a Hermitian operator are real, while non-Hermitian operators can have complex eigenvalues.

Similar threads

  • Advanced Physics Homework Help
Replies
2
Views
2K
  • Advanced Physics Homework Help
Replies
17
Views
9K
  • Advanced Physics Homework Help
Replies
2
Views
2K
  • Advanced Physics Homework Help
Replies
4
Views
1K
  • Advanced Physics Homework Help
Replies
1
Views
2K
Replies
2
Views
1K
  • Advanced Physics Homework Help
Replies
9
Views
6K
Replies
12
Views
7K
  • Advanced Physics Homework Help
Replies
17
Views
1K
  • Advanced Physics Homework Help
Replies
4
Views
3K
Back
Top